Question on sequences and induction [duplicate]












0















This question already has an answer here:




  • Show that the sequence $sqrt{2}, sqrt{2+sqrt{2}}, sqrt{2+sqrt{2+sqrt{2+}}}…$ converges and find its limit. [duplicate]

    3 answers




I'm trying to show that $a_n$ is increasing where $a_1=1$ and $a_{n+1}$ = $ sqrt{a_n+2}$



I proceeded by induction so showed that $a_{n+1}>a_n$ for all n greater than 1
So I showed it was true for n=1



Then assumed $a_{k+1}>a_k$ for some k greater than 1



Then I add two to both sides and square root to show it's true for k+1.



So $a_{k+1}+2>a_k+2$



$ sqrt{a_{k+1}+2}> sqrt{a_k+2}$



Which is $a_{k+2}>a_{k+1}$
So is this ok to show this statement?



Also how would you show that a_n has a limit and compute $lim_na_n$



I think to compute the limit you need to prove that if $b_n$ goes to b then $ sqrt{b_n+2}$ goes to $ sqrt{b+2}$ thank you for your help.










share|cite|improve this question















marked as duplicate by rtybase, Community Nov 26 at 9:54


This question has been asked before and already has an answer. If those answers do not fully address your question, please ask a new question.















  • Your argument is incorrectly structured. You prove it for $n = 1$, then, assuming it is true for $n$, show it is true for $n + 1$.
    – ncmathsadist
    Nov 26 at 1:43










  • Please try to make the titles of your questions more informative. For example, Why does $a<b$ imply $a+c<b+c$? is much more useful for other users than A question about inequality. From How can I ask a good question?: Make your title as descriptive as possible. In many cases one can actually phrase the title as the question, at least in such a way so as to be comprehensible to an expert reader. You can find more tips for choosing a good title here.
    – Shaun
    Nov 26 at 1:43






  • 1




    Please edit the question accordingly :)
    – Shaun
    Nov 26 at 1:44










  • Ok, I don't understand is that not what I have done
    – Johnf
    Nov 26 at 1:47










  • Hi, could you edit your question to make it a little clearer by including some of the steps, thanks.
    – Carlos Bacca
    Nov 26 at 2:23
















0















This question already has an answer here:




  • Show that the sequence $sqrt{2}, sqrt{2+sqrt{2}}, sqrt{2+sqrt{2+sqrt{2+}}}…$ converges and find its limit. [duplicate]

    3 answers




I'm trying to show that $a_n$ is increasing where $a_1=1$ and $a_{n+1}$ = $ sqrt{a_n+2}$



I proceeded by induction so showed that $a_{n+1}>a_n$ for all n greater than 1
So I showed it was true for n=1



Then assumed $a_{k+1}>a_k$ for some k greater than 1



Then I add two to both sides and square root to show it's true for k+1.



So $a_{k+1}+2>a_k+2$



$ sqrt{a_{k+1}+2}> sqrt{a_k+2}$



Which is $a_{k+2}>a_{k+1}$
So is this ok to show this statement?



Also how would you show that a_n has a limit and compute $lim_na_n$



I think to compute the limit you need to prove that if $b_n$ goes to b then $ sqrt{b_n+2}$ goes to $ sqrt{b+2}$ thank you for your help.










share|cite|improve this question















marked as duplicate by rtybase, Community Nov 26 at 9:54


This question has been asked before and already has an answer. If those answers do not fully address your question, please ask a new question.















  • Your argument is incorrectly structured. You prove it for $n = 1$, then, assuming it is true for $n$, show it is true for $n + 1$.
    – ncmathsadist
    Nov 26 at 1:43










  • Please try to make the titles of your questions more informative. For example, Why does $a<b$ imply $a+c<b+c$? is much more useful for other users than A question about inequality. From How can I ask a good question?: Make your title as descriptive as possible. In many cases one can actually phrase the title as the question, at least in such a way so as to be comprehensible to an expert reader. You can find more tips for choosing a good title here.
    – Shaun
    Nov 26 at 1:43






  • 1




    Please edit the question accordingly :)
    – Shaun
    Nov 26 at 1:44










  • Ok, I don't understand is that not what I have done
    – Johnf
    Nov 26 at 1:47










  • Hi, could you edit your question to make it a little clearer by including some of the steps, thanks.
    – Carlos Bacca
    Nov 26 at 2:23














0












0








0








This question already has an answer here:




  • Show that the sequence $sqrt{2}, sqrt{2+sqrt{2}}, sqrt{2+sqrt{2+sqrt{2+}}}…$ converges and find its limit. [duplicate]

    3 answers




I'm trying to show that $a_n$ is increasing where $a_1=1$ and $a_{n+1}$ = $ sqrt{a_n+2}$



I proceeded by induction so showed that $a_{n+1}>a_n$ for all n greater than 1
So I showed it was true for n=1



Then assumed $a_{k+1}>a_k$ for some k greater than 1



Then I add two to both sides and square root to show it's true for k+1.



So $a_{k+1}+2>a_k+2$



$ sqrt{a_{k+1}+2}> sqrt{a_k+2}$



Which is $a_{k+2}>a_{k+1}$
So is this ok to show this statement?



Also how would you show that a_n has a limit and compute $lim_na_n$



I think to compute the limit you need to prove that if $b_n$ goes to b then $ sqrt{b_n+2}$ goes to $ sqrt{b+2}$ thank you for your help.










share|cite|improve this question
















This question already has an answer here:




  • Show that the sequence $sqrt{2}, sqrt{2+sqrt{2}}, sqrt{2+sqrt{2+sqrt{2+}}}…$ converges and find its limit. [duplicate]

    3 answers




I'm trying to show that $a_n$ is increasing where $a_1=1$ and $a_{n+1}$ = $ sqrt{a_n+2}$



I proceeded by induction so showed that $a_{n+1}>a_n$ for all n greater than 1
So I showed it was true for n=1



Then assumed $a_{k+1}>a_k$ for some k greater than 1



Then I add two to both sides and square root to show it's true for k+1.



So $a_{k+1}+2>a_k+2$



$ sqrt{a_{k+1}+2}> sqrt{a_k+2}$



Which is $a_{k+2}>a_{k+1}$
So is this ok to show this statement?



Also how would you show that a_n has a limit and compute $lim_na_n$



I think to compute the limit you need to prove that if $b_n$ goes to b then $ sqrt{b_n+2}$ goes to $ sqrt{b+2}$ thank you for your help.





This question already has an answer here:




  • Show that the sequence $sqrt{2}, sqrt{2+sqrt{2}}, sqrt{2+sqrt{2+sqrt{2+}}}…$ converges and find its limit. [duplicate]

    3 answers








sequences-and-series






share|cite|improve this question















share|cite|improve this question













share|cite|improve this question




share|cite|improve this question








edited Nov 26 at 9:55

























asked Nov 26 at 1:38









Johnf

153




153




marked as duplicate by rtybase, Community Nov 26 at 9:54


This question has been asked before and already has an answer. If those answers do not fully address your question, please ask a new question.






marked as duplicate by rtybase, Community Nov 26 at 9:54


This question has been asked before and already has an answer. If those answers do not fully address your question, please ask a new question.














  • Your argument is incorrectly structured. You prove it for $n = 1$, then, assuming it is true for $n$, show it is true for $n + 1$.
    – ncmathsadist
    Nov 26 at 1:43










  • Please try to make the titles of your questions more informative. For example, Why does $a<b$ imply $a+c<b+c$? is much more useful for other users than A question about inequality. From How can I ask a good question?: Make your title as descriptive as possible. In many cases one can actually phrase the title as the question, at least in such a way so as to be comprehensible to an expert reader. You can find more tips for choosing a good title here.
    – Shaun
    Nov 26 at 1:43






  • 1




    Please edit the question accordingly :)
    – Shaun
    Nov 26 at 1:44










  • Ok, I don't understand is that not what I have done
    – Johnf
    Nov 26 at 1:47










  • Hi, could you edit your question to make it a little clearer by including some of the steps, thanks.
    – Carlos Bacca
    Nov 26 at 2:23


















  • Your argument is incorrectly structured. You prove it for $n = 1$, then, assuming it is true for $n$, show it is true for $n + 1$.
    – ncmathsadist
    Nov 26 at 1:43










  • Please try to make the titles of your questions more informative. For example, Why does $a<b$ imply $a+c<b+c$? is much more useful for other users than A question about inequality. From How can I ask a good question?: Make your title as descriptive as possible. In many cases one can actually phrase the title as the question, at least in such a way so as to be comprehensible to an expert reader. You can find more tips for choosing a good title here.
    – Shaun
    Nov 26 at 1:43






  • 1




    Please edit the question accordingly :)
    – Shaun
    Nov 26 at 1:44










  • Ok, I don't understand is that not what I have done
    – Johnf
    Nov 26 at 1:47










  • Hi, could you edit your question to make it a little clearer by including some of the steps, thanks.
    – Carlos Bacca
    Nov 26 at 2:23
















Your argument is incorrectly structured. You prove it for $n = 1$, then, assuming it is true for $n$, show it is true for $n + 1$.
– ncmathsadist
Nov 26 at 1:43




Your argument is incorrectly structured. You prove it for $n = 1$, then, assuming it is true for $n$, show it is true for $n + 1$.
– ncmathsadist
Nov 26 at 1:43












Please try to make the titles of your questions more informative. For example, Why does $a<b$ imply $a+c<b+c$? is much more useful for other users than A question about inequality. From How can I ask a good question?: Make your title as descriptive as possible. In many cases one can actually phrase the title as the question, at least in such a way so as to be comprehensible to an expert reader. You can find more tips for choosing a good title here.
– Shaun
Nov 26 at 1:43




Please try to make the titles of your questions more informative. For example, Why does $a<b$ imply $a+c<b+c$? is much more useful for other users than A question about inequality. From How can I ask a good question?: Make your title as descriptive as possible. In many cases one can actually phrase the title as the question, at least in such a way so as to be comprehensible to an expert reader. You can find more tips for choosing a good title here.
– Shaun
Nov 26 at 1:43




1




1




Please edit the question accordingly :)
– Shaun
Nov 26 at 1:44




Please edit the question accordingly :)
– Shaun
Nov 26 at 1:44












Ok, I don't understand is that not what I have done
– Johnf
Nov 26 at 1:47




Ok, I don't understand is that not what I have done
– Johnf
Nov 26 at 1:47












Hi, could you edit your question to make it a little clearer by including some of the steps, thanks.
– Carlos Bacca
Nov 26 at 2:23




Hi, could you edit your question to make it a little clearer by including some of the steps, thanks.
– Carlos Bacca
Nov 26 at 2:23










2 Answers
2






active

oldest

votes


















0














If
$a_{n+1}
=sqrt{a_n+2}
$

then
$a_{n+1}^2
=a_n+2
$

so
$a_{n+1}^2-a_n^2
=a_n+2-a_n^2
=-(a_n^2-a_n-2)
=-(a_n-2)(a_n+1)
$
.



Therefore,
if $0 < a_n < 2$
then
$a_{n+1}^2-a_n^2
gt 0$

since
$a_n-2 < 0$
and
$a_n+1 > 0$.



Also,
if $0 < a_n < 2$
then
$sqrt{a_n+2}
lt sqrt{4}
= 2$
.



For $a_1 = 1$
we have
$a_n$ increasing and bounded
so $a_n to L$
for some $1 < L le 2$.



Therefore
$L^2 = L+2$
so
$L = 2$
or
$L = -1$.



Since $L > 1$,
we must have
$L = 2$.



More generally,
if
$a_{n+1}
=sqrt{a_n+k}
$

then
$a_{n+1}^2
=a_n+k
$

so
$a_{n+1}^2-a_n^2
=a_n+k-a_n^2
=-(a_n^2-a_n-k)
$
.



This factors if
$1+4k$
is a square,
$(2d+1)^2 = 4k+1$
(since $4k+1$ is odd).
The roots of
$x^2-x-k = 0$
are then
$dfrac{1pm (2d+1)}{2}
=dfrac{2d+2, -2d}{2}
=d+1, -d
$
.



Therefore
$a_{n+1}^2-a_n^2
=-(a_n^2-a_n-k)
=-(a_n-d-1)(a_n+d)
$
.



If $0 < a_n le d+1$
then
$a_{n+1}^2-a_n^2
gt 0$

since
$a_n-d-1 < 0$
and
$a_n+1 > 0$.



Also,
if $0 < a_n < d+1$
since
$(2d+1)^2 = 4k+1$
so that
$k = d^2+d$,
then
$sqrt{a_n+k}
lt sqrt{d+1+d^2+d}
= sqrt{d^2+2d+1}
= d+1$
.



For $0 < a_1 lt d+1$
we have
$a_n$ increasing and bounded
so $a_n to L$
for some $1 < L le d+1$.



Therefore
$L^2 = L+k$
so
$L = d+1$
or
$L = -d$.



Since $L > 0$,
we must have
$L = d+1$.



For the original problem,
$k=2$ gives
$d=1$.






share|cite|improve this answer























  • Hi tha k you. Why do you only consider an from 0 to 2?
    – Johnf
    Nov 26 at 5:13










  • Because if $a_n > 2$ then $2 < a_{n+1} < a_n$.
    – marty cohen
    Nov 26 at 5:20












  • So does that show an is increasing or are you assuming it
    – Johnf
    Nov 26 at 5:28










  • The increasing or decreasing of $a_n$ follows from $a_{n+1}^2-a_n^2 =a_n+2-a_n^2 =-(a_n^2-a_n-2) =-(a_n-2)(a_n+1) $.
    – marty cohen
    Nov 26 at 6:53



















0














First consider that $a_n$ may have a limit $L$. If $L$ exists then $L=lim_{nto infty}a_{n+1}=lim_{nto infty}sqrt {2+a_n},=sqrt {2+L}. $



So if $a_n$ is strictly increasing with $a_1=1$ and if $L$ exists then $0<L=sqrt {2+L},$ so $L=2.$ So prove that $2>a_{n+1}>a_n >0$ for all $n$ by induction:



(i). We have $2>a_1>0.$



(ii). If $2>a_n>0$ then $4>2+a_n>0$ so $2>sqrt {2+a_n},=a_{n+1}>0$. So $2>a_n>0$ for all $n$ by induction.



(iii). Since $a_{n+1}>0$ by (ii), we have $a_{n+1}>a_niff a_{n+1}^2>a_n^2iff 2+a_n>a_n^2,$ and we do have $2+a_n>a_n^2$ when $2>a_n>0.$ So $a_{n+1}>a_n$ for all $n$ by induction.



So $a_n$ is strictly increasing and is bounded above by $2$ so it has a limit $L$. And by the first paragraph, $L=2.$



BTW, in (iii), if $2>a_n>0$ then $2+a_n>a_n+a_n=(2)(a_n)>(a_n)(a_n)=a_n^2.$






share|cite|improve this answer




























    2 Answers
    2






    active

    oldest

    votes








    2 Answers
    2






    active

    oldest

    votes









    active

    oldest

    votes






    active

    oldest

    votes









    0














    If
    $a_{n+1}
    =sqrt{a_n+2}
    $

    then
    $a_{n+1}^2
    =a_n+2
    $

    so
    $a_{n+1}^2-a_n^2
    =a_n+2-a_n^2
    =-(a_n^2-a_n-2)
    =-(a_n-2)(a_n+1)
    $
    .



    Therefore,
    if $0 < a_n < 2$
    then
    $a_{n+1}^2-a_n^2
    gt 0$

    since
    $a_n-2 < 0$
    and
    $a_n+1 > 0$.



    Also,
    if $0 < a_n < 2$
    then
    $sqrt{a_n+2}
    lt sqrt{4}
    = 2$
    .



    For $a_1 = 1$
    we have
    $a_n$ increasing and bounded
    so $a_n to L$
    for some $1 < L le 2$.



    Therefore
    $L^2 = L+2$
    so
    $L = 2$
    or
    $L = -1$.



    Since $L > 1$,
    we must have
    $L = 2$.



    More generally,
    if
    $a_{n+1}
    =sqrt{a_n+k}
    $

    then
    $a_{n+1}^2
    =a_n+k
    $

    so
    $a_{n+1}^2-a_n^2
    =a_n+k-a_n^2
    =-(a_n^2-a_n-k)
    $
    .



    This factors if
    $1+4k$
    is a square,
    $(2d+1)^2 = 4k+1$
    (since $4k+1$ is odd).
    The roots of
    $x^2-x-k = 0$
    are then
    $dfrac{1pm (2d+1)}{2}
    =dfrac{2d+2, -2d}{2}
    =d+1, -d
    $
    .



    Therefore
    $a_{n+1}^2-a_n^2
    =-(a_n^2-a_n-k)
    =-(a_n-d-1)(a_n+d)
    $
    .



    If $0 < a_n le d+1$
    then
    $a_{n+1}^2-a_n^2
    gt 0$

    since
    $a_n-d-1 < 0$
    and
    $a_n+1 > 0$.



    Also,
    if $0 < a_n < d+1$
    since
    $(2d+1)^2 = 4k+1$
    so that
    $k = d^2+d$,
    then
    $sqrt{a_n+k}
    lt sqrt{d+1+d^2+d}
    = sqrt{d^2+2d+1}
    = d+1$
    .



    For $0 < a_1 lt d+1$
    we have
    $a_n$ increasing and bounded
    so $a_n to L$
    for some $1 < L le d+1$.



    Therefore
    $L^2 = L+k$
    so
    $L = d+1$
    or
    $L = -d$.



    Since $L > 0$,
    we must have
    $L = d+1$.



    For the original problem,
    $k=2$ gives
    $d=1$.






    share|cite|improve this answer























    • Hi tha k you. Why do you only consider an from 0 to 2?
      – Johnf
      Nov 26 at 5:13










    • Because if $a_n > 2$ then $2 < a_{n+1} < a_n$.
      – marty cohen
      Nov 26 at 5:20












    • So does that show an is increasing or are you assuming it
      – Johnf
      Nov 26 at 5:28










    • The increasing or decreasing of $a_n$ follows from $a_{n+1}^2-a_n^2 =a_n+2-a_n^2 =-(a_n^2-a_n-2) =-(a_n-2)(a_n+1) $.
      – marty cohen
      Nov 26 at 6:53
















    0














    If
    $a_{n+1}
    =sqrt{a_n+2}
    $

    then
    $a_{n+1}^2
    =a_n+2
    $

    so
    $a_{n+1}^2-a_n^2
    =a_n+2-a_n^2
    =-(a_n^2-a_n-2)
    =-(a_n-2)(a_n+1)
    $
    .



    Therefore,
    if $0 < a_n < 2$
    then
    $a_{n+1}^2-a_n^2
    gt 0$

    since
    $a_n-2 < 0$
    and
    $a_n+1 > 0$.



    Also,
    if $0 < a_n < 2$
    then
    $sqrt{a_n+2}
    lt sqrt{4}
    = 2$
    .



    For $a_1 = 1$
    we have
    $a_n$ increasing and bounded
    so $a_n to L$
    for some $1 < L le 2$.



    Therefore
    $L^2 = L+2$
    so
    $L = 2$
    or
    $L = -1$.



    Since $L > 1$,
    we must have
    $L = 2$.



    More generally,
    if
    $a_{n+1}
    =sqrt{a_n+k}
    $

    then
    $a_{n+1}^2
    =a_n+k
    $

    so
    $a_{n+1}^2-a_n^2
    =a_n+k-a_n^2
    =-(a_n^2-a_n-k)
    $
    .



    This factors if
    $1+4k$
    is a square,
    $(2d+1)^2 = 4k+1$
    (since $4k+1$ is odd).
    The roots of
    $x^2-x-k = 0$
    are then
    $dfrac{1pm (2d+1)}{2}
    =dfrac{2d+2, -2d}{2}
    =d+1, -d
    $
    .



    Therefore
    $a_{n+1}^2-a_n^2
    =-(a_n^2-a_n-k)
    =-(a_n-d-1)(a_n+d)
    $
    .



    If $0 < a_n le d+1$
    then
    $a_{n+1}^2-a_n^2
    gt 0$

    since
    $a_n-d-1 < 0$
    and
    $a_n+1 > 0$.



    Also,
    if $0 < a_n < d+1$
    since
    $(2d+1)^2 = 4k+1$
    so that
    $k = d^2+d$,
    then
    $sqrt{a_n+k}
    lt sqrt{d+1+d^2+d}
    = sqrt{d^2+2d+1}
    = d+1$
    .



    For $0 < a_1 lt d+1$
    we have
    $a_n$ increasing and bounded
    so $a_n to L$
    for some $1 < L le d+1$.



    Therefore
    $L^2 = L+k$
    so
    $L = d+1$
    or
    $L = -d$.



    Since $L > 0$,
    we must have
    $L = d+1$.



    For the original problem,
    $k=2$ gives
    $d=1$.






    share|cite|improve this answer























    • Hi tha k you. Why do you only consider an from 0 to 2?
      – Johnf
      Nov 26 at 5:13










    • Because if $a_n > 2$ then $2 < a_{n+1} < a_n$.
      – marty cohen
      Nov 26 at 5:20












    • So does that show an is increasing or are you assuming it
      – Johnf
      Nov 26 at 5:28










    • The increasing or decreasing of $a_n$ follows from $a_{n+1}^2-a_n^2 =a_n+2-a_n^2 =-(a_n^2-a_n-2) =-(a_n-2)(a_n+1) $.
      – marty cohen
      Nov 26 at 6:53














    0












    0








    0






    If
    $a_{n+1}
    =sqrt{a_n+2}
    $

    then
    $a_{n+1}^2
    =a_n+2
    $

    so
    $a_{n+1}^2-a_n^2
    =a_n+2-a_n^2
    =-(a_n^2-a_n-2)
    =-(a_n-2)(a_n+1)
    $
    .



    Therefore,
    if $0 < a_n < 2$
    then
    $a_{n+1}^2-a_n^2
    gt 0$

    since
    $a_n-2 < 0$
    and
    $a_n+1 > 0$.



    Also,
    if $0 < a_n < 2$
    then
    $sqrt{a_n+2}
    lt sqrt{4}
    = 2$
    .



    For $a_1 = 1$
    we have
    $a_n$ increasing and bounded
    so $a_n to L$
    for some $1 < L le 2$.



    Therefore
    $L^2 = L+2$
    so
    $L = 2$
    or
    $L = -1$.



    Since $L > 1$,
    we must have
    $L = 2$.



    More generally,
    if
    $a_{n+1}
    =sqrt{a_n+k}
    $

    then
    $a_{n+1}^2
    =a_n+k
    $

    so
    $a_{n+1}^2-a_n^2
    =a_n+k-a_n^2
    =-(a_n^2-a_n-k)
    $
    .



    This factors if
    $1+4k$
    is a square,
    $(2d+1)^2 = 4k+1$
    (since $4k+1$ is odd).
    The roots of
    $x^2-x-k = 0$
    are then
    $dfrac{1pm (2d+1)}{2}
    =dfrac{2d+2, -2d}{2}
    =d+1, -d
    $
    .



    Therefore
    $a_{n+1}^2-a_n^2
    =-(a_n^2-a_n-k)
    =-(a_n-d-1)(a_n+d)
    $
    .



    If $0 < a_n le d+1$
    then
    $a_{n+1}^2-a_n^2
    gt 0$

    since
    $a_n-d-1 < 0$
    and
    $a_n+1 > 0$.



    Also,
    if $0 < a_n < d+1$
    since
    $(2d+1)^2 = 4k+1$
    so that
    $k = d^2+d$,
    then
    $sqrt{a_n+k}
    lt sqrt{d+1+d^2+d}
    = sqrt{d^2+2d+1}
    = d+1$
    .



    For $0 < a_1 lt d+1$
    we have
    $a_n$ increasing and bounded
    so $a_n to L$
    for some $1 < L le d+1$.



    Therefore
    $L^2 = L+k$
    so
    $L = d+1$
    or
    $L = -d$.



    Since $L > 0$,
    we must have
    $L = d+1$.



    For the original problem,
    $k=2$ gives
    $d=1$.






    share|cite|improve this answer














    If
    $a_{n+1}
    =sqrt{a_n+2}
    $

    then
    $a_{n+1}^2
    =a_n+2
    $

    so
    $a_{n+1}^2-a_n^2
    =a_n+2-a_n^2
    =-(a_n^2-a_n-2)
    =-(a_n-2)(a_n+1)
    $
    .



    Therefore,
    if $0 < a_n < 2$
    then
    $a_{n+1}^2-a_n^2
    gt 0$

    since
    $a_n-2 < 0$
    and
    $a_n+1 > 0$.



    Also,
    if $0 < a_n < 2$
    then
    $sqrt{a_n+2}
    lt sqrt{4}
    = 2$
    .



    For $a_1 = 1$
    we have
    $a_n$ increasing and bounded
    so $a_n to L$
    for some $1 < L le 2$.



    Therefore
    $L^2 = L+2$
    so
    $L = 2$
    or
    $L = -1$.



    Since $L > 1$,
    we must have
    $L = 2$.



    More generally,
    if
    $a_{n+1}
    =sqrt{a_n+k}
    $

    then
    $a_{n+1}^2
    =a_n+k
    $

    so
    $a_{n+1}^2-a_n^2
    =a_n+k-a_n^2
    =-(a_n^2-a_n-k)
    $
    .



    This factors if
    $1+4k$
    is a square,
    $(2d+1)^2 = 4k+1$
    (since $4k+1$ is odd).
    The roots of
    $x^2-x-k = 0$
    are then
    $dfrac{1pm (2d+1)}{2}
    =dfrac{2d+2, -2d}{2}
    =d+1, -d
    $
    .



    Therefore
    $a_{n+1}^2-a_n^2
    =-(a_n^2-a_n-k)
    =-(a_n-d-1)(a_n+d)
    $
    .



    If $0 < a_n le d+1$
    then
    $a_{n+1}^2-a_n^2
    gt 0$

    since
    $a_n-d-1 < 0$
    and
    $a_n+1 > 0$.



    Also,
    if $0 < a_n < d+1$
    since
    $(2d+1)^2 = 4k+1$
    so that
    $k = d^2+d$,
    then
    $sqrt{a_n+k}
    lt sqrt{d+1+d^2+d}
    = sqrt{d^2+2d+1}
    = d+1$
    .



    For $0 < a_1 lt d+1$
    we have
    $a_n$ increasing and bounded
    so $a_n to L$
    for some $1 < L le d+1$.



    Therefore
    $L^2 = L+k$
    so
    $L = d+1$
    or
    $L = -d$.



    Since $L > 0$,
    we must have
    $L = d+1$.



    For the original problem,
    $k=2$ gives
    $d=1$.







    share|cite|improve this answer














    share|cite|improve this answer



    share|cite|improve this answer








    edited Nov 26 at 5:04

























    answered Nov 26 at 4:47









    marty cohen

    72.4k549127




    72.4k549127












    • Hi tha k you. Why do you only consider an from 0 to 2?
      – Johnf
      Nov 26 at 5:13










    • Because if $a_n > 2$ then $2 < a_{n+1} < a_n$.
      – marty cohen
      Nov 26 at 5:20












    • So does that show an is increasing or are you assuming it
      – Johnf
      Nov 26 at 5:28










    • The increasing or decreasing of $a_n$ follows from $a_{n+1}^2-a_n^2 =a_n+2-a_n^2 =-(a_n^2-a_n-2) =-(a_n-2)(a_n+1) $.
      – marty cohen
      Nov 26 at 6:53


















    • Hi tha k you. Why do you only consider an from 0 to 2?
      – Johnf
      Nov 26 at 5:13










    • Because if $a_n > 2$ then $2 < a_{n+1} < a_n$.
      – marty cohen
      Nov 26 at 5:20












    • So does that show an is increasing or are you assuming it
      – Johnf
      Nov 26 at 5:28










    • The increasing or decreasing of $a_n$ follows from $a_{n+1}^2-a_n^2 =a_n+2-a_n^2 =-(a_n^2-a_n-2) =-(a_n-2)(a_n+1) $.
      – marty cohen
      Nov 26 at 6:53
















    Hi tha k you. Why do you only consider an from 0 to 2?
    – Johnf
    Nov 26 at 5:13




    Hi tha k you. Why do you only consider an from 0 to 2?
    – Johnf
    Nov 26 at 5:13












    Because if $a_n > 2$ then $2 < a_{n+1} < a_n$.
    – marty cohen
    Nov 26 at 5:20






    Because if $a_n > 2$ then $2 < a_{n+1} < a_n$.
    – marty cohen
    Nov 26 at 5:20














    So does that show an is increasing or are you assuming it
    – Johnf
    Nov 26 at 5:28




    So does that show an is increasing or are you assuming it
    – Johnf
    Nov 26 at 5:28












    The increasing or decreasing of $a_n$ follows from $a_{n+1}^2-a_n^2 =a_n+2-a_n^2 =-(a_n^2-a_n-2) =-(a_n-2)(a_n+1) $.
    – marty cohen
    Nov 26 at 6:53




    The increasing or decreasing of $a_n$ follows from $a_{n+1}^2-a_n^2 =a_n+2-a_n^2 =-(a_n^2-a_n-2) =-(a_n-2)(a_n+1) $.
    – marty cohen
    Nov 26 at 6:53











    0














    First consider that $a_n$ may have a limit $L$. If $L$ exists then $L=lim_{nto infty}a_{n+1}=lim_{nto infty}sqrt {2+a_n},=sqrt {2+L}. $



    So if $a_n$ is strictly increasing with $a_1=1$ and if $L$ exists then $0<L=sqrt {2+L},$ so $L=2.$ So prove that $2>a_{n+1}>a_n >0$ for all $n$ by induction:



    (i). We have $2>a_1>0.$



    (ii). If $2>a_n>0$ then $4>2+a_n>0$ so $2>sqrt {2+a_n},=a_{n+1}>0$. So $2>a_n>0$ for all $n$ by induction.



    (iii). Since $a_{n+1}>0$ by (ii), we have $a_{n+1}>a_niff a_{n+1}^2>a_n^2iff 2+a_n>a_n^2,$ and we do have $2+a_n>a_n^2$ when $2>a_n>0.$ So $a_{n+1}>a_n$ for all $n$ by induction.



    So $a_n$ is strictly increasing and is bounded above by $2$ so it has a limit $L$. And by the first paragraph, $L=2.$



    BTW, in (iii), if $2>a_n>0$ then $2+a_n>a_n+a_n=(2)(a_n)>(a_n)(a_n)=a_n^2.$






    share|cite|improve this answer


























      0














      First consider that $a_n$ may have a limit $L$. If $L$ exists then $L=lim_{nto infty}a_{n+1}=lim_{nto infty}sqrt {2+a_n},=sqrt {2+L}. $



      So if $a_n$ is strictly increasing with $a_1=1$ and if $L$ exists then $0<L=sqrt {2+L},$ so $L=2.$ So prove that $2>a_{n+1}>a_n >0$ for all $n$ by induction:



      (i). We have $2>a_1>0.$



      (ii). If $2>a_n>0$ then $4>2+a_n>0$ so $2>sqrt {2+a_n},=a_{n+1}>0$. So $2>a_n>0$ for all $n$ by induction.



      (iii). Since $a_{n+1}>0$ by (ii), we have $a_{n+1}>a_niff a_{n+1}^2>a_n^2iff 2+a_n>a_n^2,$ and we do have $2+a_n>a_n^2$ when $2>a_n>0.$ So $a_{n+1}>a_n$ for all $n$ by induction.



      So $a_n$ is strictly increasing and is bounded above by $2$ so it has a limit $L$. And by the first paragraph, $L=2.$



      BTW, in (iii), if $2>a_n>0$ then $2+a_n>a_n+a_n=(2)(a_n)>(a_n)(a_n)=a_n^2.$






      share|cite|improve this answer
























        0












        0








        0






        First consider that $a_n$ may have a limit $L$. If $L$ exists then $L=lim_{nto infty}a_{n+1}=lim_{nto infty}sqrt {2+a_n},=sqrt {2+L}. $



        So if $a_n$ is strictly increasing with $a_1=1$ and if $L$ exists then $0<L=sqrt {2+L},$ so $L=2.$ So prove that $2>a_{n+1}>a_n >0$ for all $n$ by induction:



        (i). We have $2>a_1>0.$



        (ii). If $2>a_n>0$ then $4>2+a_n>0$ so $2>sqrt {2+a_n},=a_{n+1}>0$. So $2>a_n>0$ for all $n$ by induction.



        (iii). Since $a_{n+1}>0$ by (ii), we have $a_{n+1}>a_niff a_{n+1}^2>a_n^2iff 2+a_n>a_n^2,$ and we do have $2+a_n>a_n^2$ when $2>a_n>0.$ So $a_{n+1}>a_n$ for all $n$ by induction.



        So $a_n$ is strictly increasing and is bounded above by $2$ so it has a limit $L$. And by the first paragraph, $L=2.$



        BTW, in (iii), if $2>a_n>0$ then $2+a_n>a_n+a_n=(2)(a_n)>(a_n)(a_n)=a_n^2.$






        share|cite|improve this answer












        First consider that $a_n$ may have a limit $L$. If $L$ exists then $L=lim_{nto infty}a_{n+1}=lim_{nto infty}sqrt {2+a_n},=sqrt {2+L}. $



        So if $a_n$ is strictly increasing with $a_1=1$ and if $L$ exists then $0<L=sqrt {2+L},$ so $L=2.$ So prove that $2>a_{n+1}>a_n >0$ for all $n$ by induction:



        (i). We have $2>a_1>0.$



        (ii). If $2>a_n>0$ then $4>2+a_n>0$ so $2>sqrt {2+a_n},=a_{n+1}>0$. So $2>a_n>0$ for all $n$ by induction.



        (iii). Since $a_{n+1}>0$ by (ii), we have $a_{n+1}>a_niff a_{n+1}^2>a_n^2iff 2+a_n>a_n^2,$ and we do have $2+a_n>a_n^2$ when $2>a_n>0.$ So $a_{n+1}>a_n$ for all $n$ by induction.



        So $a_n$ is strictly increasing and is bounded above by $2$ so it has a limit $L$. And by the first paragraph, $L=2.$



        BTW, in (iii), if $2>a_n>0$ then $2+a_n>a_n+a_n=(2)(a_n)>(a_n)(a_n)=a_n^2.$







        share|cite|improve this answer












        share|cite|improve this answer



        share|cite|improve this answer










        answered Nov 26 at 6:38









        DanielWainfleet

        34k31647




        34k31647















            Popular posts from this blog

            Quarter-circle Tiles

            build a pushdown automaton that recognizes the reverse language of a given pushdown automaton?

            Mont Emei